Inscription / Connexion Nouveau Sujet
Niveau Reprise d'études-Ter
Partager :

inégalité

Posté par
Disiz
24-07-19 à 22:40

salut

tu a une technique facile pour résoudre sa \forall x \in ] 0 ;+\infty[, \ln \left(1+\dfrac{1}{x}\right) \leqslant \dfrac{1}{\sqrt{x(x+1)}}

moi je decide de faire etude de" croissance decroissance " de une fonction ,je ne trouve pas que cest bien , toi tu fais comment?
merci

Posté par
Disiz
re : inégalité 24-07-19 à 22:51

si je fais exemple  a \leqslant \ b\leqslant c    je trouve le c puis je montre peut etre a \leqslant \ c possible?

Posté par
weierstrass
re : inégalité 24-07-19 à 23:45

Bonsoir,
Je ne sais pas si c'est le plus simple, mais en posant X = 1 + \dfrac{1}{x}, on obtient à droite \sqrt{x}-\dfrac{1}{\sqrt{x}}, ça devient plus potable à dériver

Posté par
Disiz
re : inégalité 25-07-19 à 00:12

salut ,

si je fais ce que tu dis , je ne trouve pas comme toi?
X sup à1

\ln X \leqslant \dfrac{1}{\sqrt{\dfrac{1}{X-1} \cdot \dfrac{X}{X-1}}} \Longleftrightarrow \ln X \leqslant \dfrac{X-1}{\sqrt{X}}

Posté par
Disiz
re : inégalité 25-07-19 à 00:14

desol" c est comme toi

Posté par
Disiz
re : inégalité 25-07-19 à 00:16

oui c 'est plus facile à faire  merci Karl weierstrass

Posté par
mousse42
re : inégalité 25-07-19 à 01:43

Une autre option

\ln \left(1+\dfrac{1}{x}\right) \leqslant \dfrac{1}{\sqrt{x(x+1)}} \iff \sqrt{x(x+1)}\bigg(\ln (x+1)-\ln x\bigg)\le1

En posant g(x):=\sqrt{x(x+1)}\bigg(\ln (x+1)-\ln x\bigg)

Voici deux indices :

La limite de g en 0^+

Et la monotonie de g(le produit de deux fonctions croissantes est une fonction croissante) qui se remarque sans des développements laborieux

Posté par
mousse42
re : inégalité 25-07-19 à 02:06

Désolé, oublie ce passage :

Citation :
le produit de deux fonctions croissantes est une fonction croissante


C'est vrai si elles sont à valeurs positives

Posté par
Sylvieg Moderateur
re : inégalité 25-07-19 à 09:10

Bonjour,
@mousse42,
J'ai la vague impression que x ln(x+1) - ln(x) n'est pas croissante.

@Disiz,
Dériver brutalement la différence des 2 membres initiaux pour démontrer qu'elle est décroissante ne me semble pas insurmontable.

La dérivée est du signe de 1-\frac{2x+1}{2\sqrt{x(x+1)}} .

Il reste à démontrer x+\frac{1}{2} > \sqrt{x(x+1)} ; ce qui n'est pas très difficile.

Posté par
mousse42
re : inégalité 25-07-19 à 09:17

Bonjour Sylvieg

Oui mais x\mapsto ln x -\ln (1+x) est croissante, mais à valeurs négatives d'où ma correction. Donc, je retire tout ce que j'ai proposé.

Posté par
Disiz
re : inégalité 25-07-19 à 23:16

@sylvieg

je comprend ce que tu veux dire , je fais avecf;[1 ;+\infty[\longrightarrow \mathbb{R}, X \mapsto f(X)= \dfrac{X-1}{\sqrt{X}}-\ln X pour chercher la croissance c 'est ok  les calculs sont pas dure mais c 'est longue .

@sylvieg ,mousse
je fais aussi un option sur une deuxième substitution,pour alléger le radical, le calcul est direct.t=\sqrt{X}
f :\left[1 ;+\infty\left[\longrightarrow \mathbb{R}, t \mapsto f(t)=t-\dfrac{1}{t}-2 \ln t\right.\right.
pour tout le
t \in[1 ;+\infty[,f^{\prime}(t)=\dfrac{(t-1)^{2}}{t^{2}} \geqslant 0

f est croissance sur son [1 ;+\infty[

Posté par
Disiz
re : inégalité 25-07-19 à 23:58

si tu connais quelque chose de plus fort pour résoudre le inégalité peux tu me le dire .Un ami lui me dit qui y a quelque chose de plus puissant mais il ne veut pas me dire quoi pas avant le 1er aout. donc si tu connais  ce que c 'est peux tu  faire un détailler .merci

Posté par
Sylvieg Moderateur
re : inégalité 26-07-19 à 07:40

Bonjour,
D'accord pour poser X = \sqrt{1+\frac{1}{x}}

Je ne vois pas mieux.

Posté par
lake
re : inégalité 26-07-19 à 11:44

Bonjour,

Je n'apporte rien de nouveau; juste une variante (après avoir lu ce topic):

Pour t>0,   \dfrac{2}{t}\leq 1+\dfrac{1}{t^2} que l'on intègre sur [1,u] avec u\geq 1 pour obtenir:

    \forall u\geq1,\qquad2\,\ln\,u\leq u-\dfrac{1}{u}

  Puis u=\sqrt{1+\dfrac{1}{x}}

Posté par
lake
re : inégalité 26-07-19 à 11:49

... avec x>0

Posté par
Disiz
re : inégalité 26-07-19 à 21:48

Lake tu m'as donné une idée avec le intégre...
J 'ai trouvé le résultat  de inégalité avec le Cauchy Schwarz 👍  avec le indice ^2

pour tout lex \in ] 0 ;+\infty[  ,\ln \left(1+\dfrac{1}{x}\right) \geqslant 0

so
\left(\ln \left(1+\dfrac{1}{x}\right)\right)^{2} \leqslant \dfrac{1}{x(x+1)}

substitution h=\dfrac{1}{x},

pour tout le h \geqslant 0

(\ln (1+h))^{2}=\left(\int_{1}^{1+h} \dfrac{1}{w} d w\right)^{2}\leqslant\left(\int_{1}^{1+h} 1^{2} \mathrm{d} w\right)\left(\int_{1}^{1+h} \dfrac{1}{w^{2}} \mathrm{d} w\right)=h\left(-\dfrac{1}{1+h}+1\right)=\dfrac{h^{2}}{1+h}

pour tout le x \in ] 0 ;+\infty[ , \left(\ln \left(1+\dfrac{1}{x}\right)\right)^{2} \leqslant \dfrac{\dfrac{1}{x^{2}}}{1+\dfrac{1}{x}}=\dfrac{1}{x(x+1)}  ( completed)

Posté par
lake
re : inégalité 26-07-19 à 22:25

Posté par
Sylvieg Moderateur
re : inégalité 27-07-19 à 08:31



Vous devez être membre accéder à ce service...

Pas encore inscrit ?

1 compte par personne, multi-compte interdit !

Ou identifiez-vous :


Rester sur la page

Inscription gratuite

Fiches en rapport

parmi 1675 fiches de maths

Désolé, votre version d'Internet Explorer est plus que périmée ! Merci de le mettre à jour ou de télécharger Firefox ou Google Chrome pour utiliser le site. Votre ordinateur vous remerciera !